Psychiatric Nursing Chapter 16: Depressive Disorders, Ch. 11 - Suicidal Thoughts and Behavior, Chapter 13 Neurocognitive disorder, Chapter 15: Schizophrenia and Other Psychotic Disorders NCLEX, Chapter 15: Schizophrenia Spectrum and Other Psychotic D...

Lakukan tugas rumah & ujian kamu dengan baik sekarang menggunakan Quizwiz!

An elderly patient is admitted with delirium secondary to a urinary tract infection. The family asks whether the patient will ever recover. Select the nurse's best response? A: "The health care provider is the best person to answer your question". B: "The confusion will probably get better as we treat the infection". C: "Unfortunately, delirium is a progressively disabling disorder". D: "I will be glad to contact the chaplain to talk with you.

"The confusion will probably get better as we treat the infection".

21. ______________ are false sensory perceptions not associated with real external stimuli and may involve any of the five senses.

ANS: Hallucinations Page: 352 Feedback: Hallucinations are false sensory perceptions not associated with real external stimuli and may involve any of the five senses. Types of hallucinations include auditory, visual, tactile, gustatory, and olfactory.

If clozapine (Clozaril) therapy is being considered, the nurse should evaluate which laboratory test to establish a baseline for comparison in order to recognize a potentially life-threatening side effect? A. White blood cell count B. Liver function studies C. Creatinine clearance D. Blood urea nitrogen

A (The nurse should establish a baseline white blood cell count to evaluate a potentially life-threatening side effect if clozapine (Clozaril) is being considering as a treatment option. Clozapine can have a serious side effect of agranulocytosis in which a potentially fatal drop in white blood cells can occur.)

A client diagnosed with major NCD is exhibiting behavioral problems on a daily basis. At change of shift, the client's behavior escalates from pacing to screaming and flailing. Which action should be a nursing priority? A: Consult the psychologist regarding behavior-modification techniques. B: Medicate the client with prn antianxiety medications. C: Assess environmental triggers and potential unmet needs. D: Anticipate the behavior and restrain when pacing begins.

2: Medicate the client with prn antianxiety medications.

Consider these diagnostic findings: apolipoprotein E (apoE) malfunction, neurofibrillary tangles, neuronal degeneration in the hippocampus, and brain atrophy. Which health problem corresponds to these diagnostic findings? a. Huntington's disease b. Alzheimer's disease c. Parkinson's disease d. Vascular dementia

B: Alzheimer's disease

What is the priority nursing intervention for a patient diagnosed with delirium who has fluctuating levels of consciousness, disturbed orientation, and perceptual alterations? A: Distraction using sensory stimulation B: Careful observation and supervision C: Avoidance of physical contact D: Activation of the bed alarm.

B: Careful observation and supervision

After taking chlorpromazine (Thorazine) for 1 month, a client presents to an emergency department (ED) with severe muscle rigidity, tachycardia, and a temperature of 105oF (40.5C). The nurse expects the physician to recognize which condition and implement which treatment? A. Neuroleptic malignant syndrome and treat by discontinuing Thorazine and administering dantrolene (Dantrium) B. Neuroleptic malignant syndrome and treat by increasing Thorazine dosage and administering an antianxiety medication C. Dystonia and treat by administering trihexyphenidyl (Artane) D. Dystonia and treat by administering bromocriptine (Parlodel)

A (The nurse should expect that an ED physician would diagnose the client with neuroleptic malignant syndrome and treat the client by discontinuing chlorpromazine (Thorazine) and administering dantrolene (Dantrium). Neuroleptic malignant syndrome is a potentially fatal condition characterized by muscle rigidity, fever, altered consciousness, and autonomic instability. The use of typical antipsychotics is largely being replaced by atypical antipsychotics due to fewer side effects and lower risks.)

A 16-year-old-client diagnosed with paranoid schizophrenia experiences command hallucinations to harm others. The client's parents ask a nurse, "Where do the voices come from?" Which is the appropriate nursing reply? A. "Your child has a chemical imbalance of the brain which leads to altered thoughts." B. "Your child's hallucinations are caused by medication interactions." C. "Your child has too little serotonin in the brain causing delusions and hallucinations." D. "Your child's abnormal hormonal changes have precipitated auditory hallucinations."

A (The nurse should explain that a chemical imbalance of the brain leads to altered thought processes. Hallucinations, or false sensory perceptions, may occur in all five senses. The client who hears voices is experiencing an auditory hallucination.)

A client diagnosed with schizophrenia is prescribed clozapine (Clozaril). Which client symptoms related to the side effects of this medication should prompt a nurse to intervene immediately? A. Sore throat, fever, and malaise B. Akathisia and hypersalivation C. Akinesia and insomnia D. Dry mouth and urinary retention

A (The nurse should intervene immediately if the client experiences a sore throat, fever, and malaise when taking the atypical antipsychotic drug clozapine (Clozaril). Clozapine can have a serious side effect of agranulocytosis, in which a potentially fatal drop in white blood cells can occur. Symptoms of infectious processes would alert the nurse to this potential.)

Which statement should indicate to a nurse that an individual is experiencing a delusion? A. "There's an alien growing in my liver." B. "I see my dead husband everywhere I go." C. "The IRS may audit my taxes." D. "I'm not going to eat my food. It smells like brimstone."

A (The nurse should recognize that a client who claims that an alien is inside his or her body is experiencing a delusion. Delusions are false personal beliefs that are inconsistent with the person's intelligence or cultural background.)

Laboratory results reveal decreased levels of prolactin in a client diagnosed with schizophrenia. When assessing the client, which symptoms should a nurse expect to observe?(Select all that apply.) A. Apathy B. Social withdrawal C. Anhedonia D. Auditory hallucinations E. Delusions

A, B, C (The nurse should expect that a client with decreased levels of prolactin would experience apathy, social withdrawal, and anhedonia. Decreased levels of prolactin can cause depression which would result in the above symptoms.)

The diagnosis of catatonic disorder associated with another medical condition is made when the client's medical history, physical examination, or laboratory findings provide evidence that symptoms are directly attributed to which of the following? (Select all that apply.) A. Hyperthyroidism B. Hypothyroidism C. Hyperadrenalism D. Hypoadrenalism E. Hyperaphia

A, B, C, D (The diagnosis of catatonic disorder associated with another medical condition is made when the symptomatology is evidenced from medical history, physical examination, or laboratory findings to be directly attributable to the physiological consequences of a general medical condition. Types of medical conditions that have been associated with catatonic disorder include metabolic disorders (e.g., hepatic encephalopathy, hypo- and hyperthyroidism, hypo- and hyperadrenalism, and vitamin B12 deficiency) and neurological conditions (e.g., epilepsy, tumors, cerebrovascular disease, head trauma, and encephalitis). Hyperaphia is an excessive sensitivity to touch.)

Which of the following components should a nurse recognize as an integral part of a rehabilitative program when planning care for clients diagnosed with schizophrenia? (Select all that apply.) A. Group therapy B. Medication management C. Deterrent therapy D. Supportive family therapy E. Social skills training

A, B, D, E (The nurse should recognize that group therapy, medication management, supportive family therapy, and social skills training all play an integral part in rehabilitative programs for clients diagnosed with schizophrenia. Schizophrenia results from various combinations of genetic predispositions, biochemical dysfunctions, physiological factors, and psychological stress. Effective treatment requires a comprehensive, multidisciplinary effort.)

A nurse is administering risperidone (Risperdal) to a client diagnosed with schizophrenia. The therapeutic effect of this medication would most effectively address which of the following symptoms? (Select all that apply.) A. Somatic delusions B. Social isolation C. Gustatory hallucinations D. Flat affect E. Clang associations

A, C, E (The nurse should expect that risperidone (Risperdal) would be effective treatment for somatic delusions, gustatory hallucinations, and clang associations. Risperidone is an atypical antipsychotic that has been effective in the treatment of the positive symptoms of schizophrenia and in maintenance therapy to prevent exacerbation of schizophrenic symptoms.)

A geriatric nurse is teaching the clients family about the possible cause of delirium. Which statement by the nurse is most accurate? A: "Taking multiple medications may lead to adverse interactions or toxicity" B: "Age-related cognitive changes may lead to alterations in mental status". C: "Lack of rigorous exercise may lead to decreased cerebral blood flow". D: "Decreased social interaction may lead to profound isolation and psychosis".

A: "Taking multiple medications may lead to adverse interactions or toxicity".

A client diagnosed recently with AD is prescribed donepezil (Aricept). the client's spouse inquires, "How does this work? Will this cure him?" Which is the appropriate nursing response? A: "this medication delays the destruction of acetlycholine, a chemical in the brain necessary for memory process. Although most effective in the early stages, it serves to delay, but not stop, the progression of the disease" B: "This medication encourages production of acetlycholine, a chemical in the brain necessary for memory processes. It delays the progression of the disease" C: "This medication delays the destruction of dopamine, a chemical in the brain necessary for memory processes. Although most effective in the early stages, it serves to delay, but not stop, the progression of the disease". D: "This medication encourages production of dopamine, a chemical in the brain necessary for memory processes. It delays the progression of the disease".

A: "this medication delays the destruction of acetlycholine, a chemical in the brain necessary for memory process. Although most effective in the early stages, it serves to delay, but not stop, the progression of the disease".

During morning care, a nurse asks a patient diagnosed with dementia, "How was your night?" The patient replies, "It was lovely. I went out to dinner and a movie with my friend." Which term applies to the patient's response? A: Sundown syndrome B: Confabulation C: Perseveration D: Delirium

B: Confabulation

An elderly person presents with symptoms of delirium. The family reports, "Everything was fine until yesterday." What is the most important assessment information for the nurse to gather? A: A list of all medications the person currently takes. B: Whether the person has experienced any recent losses. C: Whether the person has ingested ages or fermented foods. D: The person's recent personality characteristics and changes

A: A list of all medications the person currently takes.

Which of the following medications that have been known to precipitate delirium? (Select all that apply) A: Antineoplastic agents B: H2-receptor antagonists C: Antihypertensives D: Corticosteroids E: Lipid-lowering agents

A: Antineoplastic agents B: H2-receptor antagonists C: Antihypertensives D: corticosteroids

A patient has progressive memory deficits associated with dementia. Which nursing intervention would best help the individual function in the environment? a. Assist the patient to perform simple tasks by giving step-by-step directions. b. Reduce frustration by performing activities of daily living for the patient. c. Stimulate intellectual function by discussing new topics with the patient. d. Read one story from the newspaper to the patient every day.

A: Assist the patient to perform simple tasks by giving step-by-step directions.

An older adult patient takes multiple medications daily. Over 2 days, the patient developed confusion, slurred speech, an unsteady gait, and fluctuating levels of orientation. these findings are most characteristic of? A: Delirium B: Dementia C: Amnestic syndrome D: Alzheimer's disease

A: Delirium

An older adult is prescribed digoxin (Lanoxin) and hydrochlorothiazide daily as well as lorazepam (Ativan) as needed for anxiety. Over 2 days, the patient developed confusion, slurred speech, an unsteady gait, and fluctuating levels of orientation. What is the most likely reason for the patient's change in mental status? A: Drug actions and interactions B: Benzodiazepine withdrawal C: Hypotensive episodes D: Renal failure

A: Drug actions and interactions

A husband has agreed to admit his spouse, diagnosed with Alzheimer's disease (AD), to a long-term care facility. He is expressing feelings of guilt and symptoms of depression. Which appropriate nursing diagnosis and subsequent intervention would the nurse document? A: Dysfunctional grieving; AD support group B: Altered thought process; AD support group C: Major depressive episode; psychiatric referral D: Caregiver role strain; psychiatric referral

A: Dysfunctional grieving; AD support group.

Which of the follwoing conditions have been known to precipitate delirium in some individuals? (Select all that apply) A: Febrile illness B: Seizures C: Migraine headaches D: Herniated brain stem E: Temporomandibular joint syndrome.

A: Febrile illness B: Seizures C: Migraine headaches

Which assessment findings would the nurse expect in a patient experiencing delirium? Select all that apply: A: Impaired level of consciousness B: Disorientation to place, time C: Wandering attention D: Apathy E: Agnosia

A: Impaired level of consciousness B: Disorientation to place, time. C: Wandering attention

An older adult with moderately severe dementia forgets where the bathroom is and has episodes of incontinence. Which intervention should the nurse suggest to the patient's family? A: Label the bathroom door. B: Take the older adult to the bathroom hourly. C: Place the older adult in disposable adult briefs D: Limit the intake of oral fluids to 1000 ml per day

A: Label the bathroom door

A patient diagnosed with delirium is experiencing perceptual alterations. Which environmental adjustment should the nurse make for this patient? A: Provide a well-lit room without glare or shadows. Limit noise and stimulation. B: Maintain soft lighting day and night. Keep a radio on low volume continuously C: Light the room brightly day and night. Awaken the patient hourly to assess mental status. D: Keep the patient by the nurse's desk while awake. Provide rest periods in a room with a television on.

A: Provide a well-lit room without glare or shadows. Limit noise and stimulation.

A patient diagnosed with moderately severe Alzheimer's disease has a self-care deficit of dressing and grooming. Designate appropriate interventions to include in the patient's plan of care. Select all that apply: A: Provide clothing with elastic and hook-and-loop closures B: Label clothing with the patient's name and name of the item. C: Administer anti-anxiety medication before bathing and dressing. D: Provide necessary items and direct the patient to proceed independently E: If the patient resists dressing, use distraction and try again after a short interval

A: Provide clothing with elastic and hook-and-loop closures B: Label clothing with the patient's name and name of the item. E: If the patient resists dressing, use distraction and try again after a short interval

What is the priority nursing diagnosis for a patient with fluctuating levels of consciousness, disturbed orientation, and visual and tactile hallucinations? A: Risk for injury related to altered cerebral function, fluctuating levels of consciousness, disturbed orientation, and misperception of the environment. B: Bathing/hygiene self-care deficit related to cerebral dysfunction, as evidenced by confusion and inability to perform personal hygiene tasks. C: Disturbed thought processes related to medication intoxication, as evidenced by confusion, disorientation, and hallucinations. D: Fear related to sensory perceptual alterations as evidenced by visual and tactile hallucinations.

A: Risk for injury related to altered cerebral function, fluctuating levels of consciousness, disturbed orientation, and misperception of the environment.

An older adult patient in the intensive care unit has visual and auditory illusions. Which intervention will be most helpful? A: Using the patient's glasses and hearing aids B: Placing personally meaningful objects in view C: Placing large clocks and calendars on the wall D: Assuring that the room is brightly lit but very quiet at all times.

A: Using the patient's glasses and hearing aids

Goals of care for an older adult patient diagnosed with delirium caused by fever and dehydration will focus on: a. returning to premorbid levels of function. b. identifying stressors negatively affecting self. c. demonstrating motor responses to noxious stimuli. d. exerting control over responses to perceptual distortions.

A: returning to premorbid levels of function

Consider these health problems: Lew body disease, frontal-temporal lobar degeneration, and Huntington's disease. Which term unifies these problems? A: Cyclothymia B: Dementia C: Delirium D: Amnesia

B: Dementia

20. _____________________ disorder is manifested by schizophrenic behaviors, with a strong element of symptomatology associated with the mood disorders (depression or mania).

ANS: Schizoaffective Page: 349 Feedback: Schizoaffective disorder is manifested by schizophrenic behaviors, with a strong element of symptomatology associated with the mood disorders (depression or mania). The decisive factor in the diagnosis of schizoaffective disorder is the presence of hallucinations or delusions that occur for at least 2 weeks in the absence of a major mood episode.

19. Order the spectrum of schizophrenic and other psychotic disorders as described by the DSM-5 on a gradient of psychopathology from least to most severe. ________ Delusional disorder ________ Schizotypal personality disorder ________ Schizophrenia ________ Brief psychotic disorder ________ Psychotic disorder associated with another medical condition ________ Catatonic disorder associated with another medical condition ________ Schizoaffective disorder ________ Schizophreniform disorder ________ Substance-induced psychotic disorder

ANS: The correct order is 2, 1, 9, 3, 5, 6, 8, 7, 4 Page: 347-348 Feedback: A spectrum of schizophrenic and other psychotic disorders has been identified in the DSM-5. These include (on a gradient of psychopathology from least to most severe): schizotypal personality disorder, delusional disorder, brief psychotic disorder, substance-induced psychotic disorder, psychotic disorder associated with another medical condition, catatonic disorder associated with another medical condition, schizophreniform disorder, schizoaffective disorder, and schizophrenia. 1. Schizotypal personality disorder 2. Delusional disorder 3. Brief psychotic disorder 4. Substance-induced psychotic disorder 5. Psychotic disorder associated with another medical condition 6. Catatonic disorder associated with another medical condition 7. Schizophreniform disorder 8. Schizoaffective disorder 9. Schizophrenia

9. A nurse reviews the laboratory data of a client suspected of having the diagnosis of major depressive episode. Which lab value would potentially rule out this diagnosis? 1. Thyroid-stimulating hormone (TSH) level of 25 U/mL 2. Potassium (K+) level of 4.2 mEq/L 3. Sodium (Na+) level of 140 mEq/L 4. Calcium (Ca2+) level of 9.5 mg/dL

ANS: 1 Rationale: A diagnosis of major depressive episode may be ruled out if the client's lab results reveal a TSH level of 25 U/mL. Normal levels of TSH range from 2 to 10 U/mL. High levels of TSH indicate low thyroid function. The client's high TSH value may indicate hypothyroidism, which can lead to depressive symptoms. The DSM-5 criteria for the diagnosis of major depressive episode states that this diagnosis must not be attributable to the direct physiological effects of another medical condition.

20. A client, who is taking transdermal selegiline (Emsam) for depressive symptoms, states, "My physician told me there was no need to worry about dietary restrictions." Which would be the most appropriate nursing response? 1. "Because your dose of Emsam is 6 mg in 24 hours, dietary restrictions are not recommended." 2. "You must have misunderstood. An MAOI like Emsam always has dietary restrictions." 3. "Only oral MAOIs require dietary restrictions." 4. "All transdermal MAOIs do not require dietary modifications."

ANS: 1 Rationale: Selegiline is a Monoamine Oxidase Inhibitor (MAOI). Hypertensive crisis, caused by the ingestion of foods high in tyramine, has not shown to be a problem with selegiline transdermal system at the 6 mg/24 hr dosage, and dietary restrictions at this dose are not recommended. Dietary modifications are recommended, however, at the 9 mg/24 hr and 12 mg/24 hr dosages.

2. A husband has agreed to admit his spouse, diagnosed with Alzheimer's disease (AD), to a long-term care facility. He is expressing feelings of guilt and symptoms of depression. Which appropriate nursing diagnosis and subsequent intervention would the nurse document? 1. Dysfunctional grieving; AD support group 2. Altered thought process; AD support group 3. Major depressive episode; psychiatric referral 4. Caregiver role strain; psychiatric referral

ANS: 1 Rationale: The most appropriate nursing diagnosis and intervention for the husband is dysfunctional grieving; AD support group. Clients with AD are often at risk for trauma and have significant self-care deficits that require more care than a spouse may be able to provide.

7. A client diagnosed recently with AD is prescribed donepezil (Aricept). The client's spouse inquires, "How does this work? Will this cure him?" Which is the appropriate nursing response? 1. "This medication delays the destruction of acetylcholine, a chemical in the brain necessary for memory processes. Although most effective in the early stages, it serves to delay, but not stop, the progression of the disease." 2. "This medication encourages production of acetylcholine, a chemical in the brain necessary for memory processes. It delays the progression of the disease." 3. "This medication delays the destruction of dopamine, a chemical in the brain necessary for memory processes. Although most effective in the early stages, it serves to delay, but not stop, the progression of the disease." 4. "This medication encourages production of dopamine, a chemical in the brain necessary for memory processes. It delays the progression of the disease."

ANS: 1 Rationale: The most appropriate response by the nurse is to explain that donepezil delays the destruction of acetylcholine, a chemical in the brain necessary for memory processes. Although most effective in the early stages, it serves to delay, but not stop, the progression of the AD. Some side effects include dizziness, headache, gastrointestinal upset, and elevated transaminase.

1. A geriatric nurse is teaching the client's family about the possible cause of delirium. Which statement by the nurse is most accurate? 1. "Taking multiple medications may lead to adverse interactions or toxicity." 2. "Age-related cognitive changes may lead to alterations in mental status." 3. "Lack of rigorous exercise may lead to decreased cerebral blood flow." 4. "Decreased social interaction may lead to profound isolation and psychosis."

ANS: 1 Rationale: The nurse should identify that taking multiple medications that may lead to adverse reactions or toxicity is a risk factor for the development of delirium in older adults. Symptoms of delirium include difficulty sustaining and shifting attention. The client with delirium is disoriented to time and place and may also have impaired memory.

4. Immediately after electroconvulsive therapy (ECT), in which position should a nurse place the client? 1. On his or her side, to prevent aspiration 2. In high Fowler's position, to prevent increased intracranial pressure 3. In Trendelenburg's position, to promote blood flow to vital organs 4. In prone position, to prevent airway blockage

ANS: 1 Rationale: The nurse should place a client who has received ECT on his or her side, to prevent aspiration.

18. Which of the following conditions have been known to precipitate delirium in some individuals? (Select all that apply.) 1. Febrile illness 2. Seizures 3. Migraine headaches 4. Herniated brain stem 5. Temporomandibular joint syndrome

ANS: 1, 2, 3 Rationale: Delirium most commonly occurs in individuals with serious medical, surgical, or neurological conditions. Some examples of conditions that have been known to precipitate delirium in some individuals include the following: systemic infections; febrile illness; metabolic disorders, such as hypoxia, hypercarbia, or hypoglycemia; hepatic encephalopathy; head trauma; seizures; migraine headaches; brain abscess; stroke; postoperative states; and electrolyte imbalance. A herniated brain stem would most likely result in death, not delirium. Temporomandibular joint syndrome is marked by limited movement of the joint during chewing, not delirium.

25. A nursing instructor is teaching about the new DSM-5 diagnostic category of disruptive mood dysregulation disorder (DMDD). Which of the following information should the instructor include? (Select all that apply.) 1. Symptoms include verbal rages or physical aggression toward people or property. 2. Temper outbursts must be present in at least two settings (at home, at school, or with peers). 3. DMDD is characterized by severe recurrent temper outbursts. 4. The temper outbursts are manifested only behaviorally. 5. Symptoms of DMDD must be present for 18 or more months to meet diagnostic criteria.

ANS: 1, 2, 3 Rationale: The APA has included a new diagnostic category in the Depressive Disorders chapter of the DSM-5. This childhood disorder is called disruptive mood dysregulation disorder. Criteria for the diagnosis include, but are not limited to, the following. Verbal rages or physical aggression toward people or property; temper outbursts must be present in at least two settings (at home, at school, or with peers). DMDD is characterized by severe recurrent temper outbursts. The temper outbursts are manifested both behaviorally and/or verbally. Symptoms of DMDD must be present for 12, not 18 or more months to meet diagnostic criteria.

Laboratory results reveal decreased levels of prolactin in a client diagnosed with schizophrenia spectrum disorder. Which of the following client symptoms would a nurse expect to observe during assessment? (Select all that apply.) 1. Apathy 2. Social withdrawal 3. Anhedonia 4. Auditory hallucinations 5. Delusions

ANS: 1, 2, 3 Rationale: The nurse should expect that a client with decreased levels of prolactin may experience apathy, social withdrawal, and anhedonia. Decreased levels of prolactin can cause depression, which could result in these symptoms.

18. The diagnosis of catatonic disorder associated with another medical condition is made when the client's medical history, physical examination, or laboratory findings provide evidence that symptoms are directly attributed to which of the following? (Select all that apply.) 1. Hyperthyroidism 2. Hypothyroidism 3. Hyperadrenalism 4. Hypoadrenalism 5. Hyperaphia

ANS: 1, 2, 3, 4 Page: 348-349 Feedback 1. The diagnosis of catatonic disorder associated with another medical condition is made when the symptomatology is evidenced from medical history, physical examination, or laboratory findings to be directly attributable to the physiological consequences of a general medical condition. Types of medical conditions that have been associated with catatonic disorder include metabolic disorders such as hyperthyroidism. 2. The diagnosis of catatonic disorder associated with another medical condition is made when the symptomatology is evidenced from medical history, physical examination, or laboratory findings to be directly attributable to the physiological consequences of a general medical condition. Types of medical conditions that have been associated with catatonic disorder include metabolic disorders, such as hypothyroidism. 3. The diagnosis of catatonic disorder associated with another medical condition is made when the symptomatology is evidenced from medical history, physical examination, or laboratory findings to be directly attributable to the physiological consequences of a general medical condition. Types of medical conditions that have been associated with catatonic disorder include metabolic disorders, such as hyperadrenalism. 4. The diagnosis of catatonic disorder associated with another medical condition is made when the symptomatology is evidenced from medical history, physical examination, or laboratory findings to be directly attributable to the physiological consequences of a general medical condition. Types of medical conditions that have been associated with catatonic disorder include metabolic disorders, such as hypoadrenalism. 5. Hyperaphia is an excessive sensitivity to touch.

19. Which of the following medications that have been known to precipitate delirium? (Select all that apply.) 1. Antineoplastic agents 2. H2-receptor antagonists 3. Antihypertensives 4. Corticosteroids 5. Lipid-lowering agents

ANS: 1, 2, 3, 4 Rationale: Medications that have been known to precipitate delirium include anticholinergics, antihypertensives, corticosteroids, anticonvulsants, cardiac glycosides, analgesics, anesthetics, antineoplastic agents, antiparkinson drugs, H2-receptor antagonists (e.g., cimetidine), and others. There have been no reports of delirium ascribed to the use of lipid-lowering agents.

The diagnosis of catatonic disorder associated with another medical condition is made when the client's medical history, physical examination, or laboratory findings provide evidence that symptoms are directly attributed to which of the following? (Select all that apply.) 1. Hyperthyroidism2. Hypothyroidism 3. Hyperadrenalism 4. Hypoadrenalism 5. Hyperaphia

ANS: 1, 2, 3, 4 Rationale: The diagnosis of catatonic disorder associated with another medical condition is made when the symptomatology is evidenced from medical history, physical examination, or laboratory findings to be directly attributable to the physiological consequences of a general medical condition. Types of medical conditions that have been associated with catatonic disorder include metabolic disorders (e.g., hepatic encephalopathy, hypo- and hyperthyroidism, hypo- and hyperadrenalism, and vitamin B12 deficiency) and neurological conditions (e.g., epilepsy, tumors, cerebrovascular disease, head trauma, and encephalitis). Hyperaphia is an excessive sensitivity to touch

24. A client is prescribed phenelzine (Nardil). Which of the following statements by the client should indicate to a nurse that discharge teaching about this medication has been successful? (Select all that apply.) 1. "I'll have to let my surgeon know about this medication before I have my cholecystectomy." 2. "I guess I will have to give up my glass of red wine with dinner." 3. "I'll have to be very careful about reading food and medication labels." 4. "I'm going to miss my caffeinated coffee in the morning." 5. "I'll be sure not to stop this medication abruptly."

ANS: 1, 2, 3, 5 Rationale: The nurse should evaluate that teaching has been successful when the client states that phenelzine should not be taken in conjunction with the use of alcohol or foods high in tyramine and should not be stopped abruptly. Phenelzine is an MAOI that can have negative interaction with other medications. The client needs to tell other physicians about taking MAOIs, because of the risk of drug interactions.

17. Which of the following components should a nurse recognize as an integral part of a rehabilitative program when planning care for clients diagnosed with schizophrenia spectrum disorder? (Select all that apply.) 1. Group therapy 2. Medication management 3. Deterrent therapy 4. Supportive family therapy 5. Social skills training

ANS: 1, 2, 4, 5 Page: 365-368 Feedback 1. The nurse should recognize that group therapy plays an integral part of rehabilitative programs for clients diagnosed with schizophrenia spectrum disorder. 2. The nurse should recognize that medication management plays an integral part of rehabilitative programs for clients diagnosed with schizophrenia spectrum disorder. 3. Deterrent therapy is not a part of rehabilitative programs. 4. The nurse should recognize that supportive family therapy plays an integral part of rehabilitative programs for clients diagnosed with schizophrenia spectrum disorder. 5. The nurse should recognize that social skills training plays an integral part of rehabilitative programs for clients diagnosed with schizophrenia spectrum disorder.

Which of the following components should a nurse recognize as an integral part of a rehabilitative program when planning care for clients diagnosed with schizophrenia spectrum disorder? (Select all that apply.) 1. Group therapy 2. Medication management 3. Deterrent therapy 4. Supportive family therapy 5. Social skills training

ANS: 1, 2, 4, 5 Rationale: The nurse should recognize that group therapy, medication management, supportive family therapy, and social skills training all play an integral part of rehabilitative programs for clients diagnosed with schizophrenia spectrum disorder. Schizophrenia results from various combinations of genetic predispositions, biochemical dysfunctions, physiological factors, and psychological stress. Effective treatment requires a comprehensive, multidisciplinary effort

A nurse is administering risperidone (Risperdal) to a client diagnosed with schizophrenia spectrum disorder. Which of the following client symptoms would most likely decrease because of the therapeutic effect of this medication? (Select all that apply.) 1. Somatic delusions 2. Social isolation 3. Gustatory hallucinations 4. Flat affect 5. Clang associations

ANS: 1, 3, 5 Rationale: The nurse should expect that risperidone would be effective treatment for the positive symptoms of somatic delusions, gustatory hallucinations, and clang associations. Risperidone is an atypical antipsychotic that has been effective in the treatment of the positive symptoms of schizophrenia and in maintenance therapy to prevent exacerbation of schizophrenic symptoms

22. A nursing home resident has a diagnosis of dysthymic disorder. When planning care for this client, which of the following symptoms should a nurse expect the client to exhibit? (Select all that apply.) 1. Sad mood on most days 2. Mood rating of 2 out of 10 for the past 6 months 3. Labile mood 4. Sad mood for the past 3 years after spouse's death 5. Pressured speech when communicating

ANS: 1, 4 Rationale: The nurse should anticipate that a client with a diagnosis of dysthymic disorder would experience a sad mood on most days for more than two years. The essential feature of dysthymia is a chronically depressed mood, which can have an early or late onset.

17. A number of assessment rating scales are available for measuring severity of depressive symptoms. Which scale would a nurse practitioner use to assess a depressed client? 1. Zung Depression Scale 2. Hamilton Depression Rating Scale 3. Beck Depression Inventory 4. AIMS Depression Rating Scale

ANS: 2 Rationale: A number of assessment rating scales are available for measuring severity of depressive symptoms. Some are meant to be clinician administered, whereas others may be self-administered. Examples of self-rating scales include the Zung Self-Rating Depression Scale and the Beck Depression Inventory. One of the most widely used clinician-administered scales is the Hamilton Depression Rating Scale. The Abnormal Involuntary Movement Scale (AIMS) is a rating scale that measures involuntary movements associated with tardive dyskinesia.

5. A client is diagnosed with major depressive episode. Which nursing diagnosis should a nurse assign to this client, to address a behavioral symptom of this disorder? 1. Altered communication R/T feelings of worthlessness AEB anhedonia 2. Social isolation R/T poor self-esteem AEB secluding self in room 3. Altered thought processes R/T hopelessness AEB persecutory delusions 4. Altered nutrition: less than body requirements R/T high anxiety AEB anorexia

ANS: 2 Rationale: A nursing diagnosis of social isolation R/T poor self-esteem AEB secluding self in room addresses a behavioral symptom of major depressive episode. Other behavioral symptoms include psychomotor retardation, virtually nonexistent communication, curled-up position, and no attention to personal hygiene and grooming.

21. After 6 months of taking imipramine (Tofranil) for depressive symptoms, a client complains that the medication doesn't seem as effective as before. Which question should the nurse ask to determine the cause of this problem? 1. "Are you consuming foods high in tyramine?" 2. "How many packs of cigarettes do you smoke daily?" 3. "Do you drink any alcohol?" 4. "Are you taking St. John's wort?"

ANS: 2 Rationale: Imipramine is a tricyclic antidepressant. Smoking should be avoided while receiving tricyclic therapy. Smoking increases the metabolism of tricyclics, requiring an adjustment in dosage to achieve the therapeutic effect. Alcohol potentiates the effects of antidepressants. Tyramine is only an issue when MAOI medications are prescribed. Concomitant use of St. John's wort and SSRIs, not tricyclics, increases, not decreases the effects of the drug.

17. Which statement accurately differentiates NCD from pseudodementia (depression)? 1. NCD has a rapid onset, whereas pseudodementia does not. 2. NCD symptoms include disorientation to time and place, and pseudodementia does not. 3. NCD symptoms improve as the day progresses, but symptoms of pseudodementia worsen. 4. NCD causes decreased appetite, whereas pseudodementia does not.

ANS: 2 Rationale: NCD has a slow progression of symptoms, whereas pseudodementia has a rapid progression of symptoms. NCD symptoms include disorientation to time and place, and pseudodementia does not. NCD symptoms' severity worsens as the day progresses, whereas in pseudodementia, symptoms improve as the day progresses. In NCD the appetite remains unchanged. whereas in pseudodementia, the appetite diminishes.

A nurse prepares to assess a client using the Abnormal Involuntary Movement Scale (AIMS). Which side effect of antipsychotic medications led to the use of this assessment tool? 1. Dystonia 2. Tardive dyskinesia 3. Akinesia 4. Akathisia

ANS: 2 Rationale: The AIMS is a rating scale that was developed in the 1970s by the National Institute of Mental Health to measure involuntary movements associated with tardive dyskinesia

14. A nurse recently admitted a client to an inpatient unit after a suicide attempt. A health-care provider orders amitriptyline (Elavil) for the client. Which intervention, related to this medication, should be initiated to maintain this client's safety upon discharge? 1. Provide a 6-month supply of Elavil to ensure long-term compliance. 2. Provide a 1-week supply of Elavil, with refills contingent on follow-up appointments. 3. Provide pill dispenser as a memory aid. 4. Provide education regarding the avoidance of foods containing tyramine.

ANS: 2 Rationale: The health-care provider should provide no more than a 1-week supply of amitriptyline, with refills contingent on follow-up appointments, as an appropriate intervention to maintain the client's safety. Antidepressants, which are central nervous system depressants, can be used to commit suicide. Also these medications can precipitate suicidal thoughts during the initial use period. Limiting the amount of medication and monitoring the client weekly would be appropriate interventions to address the client's risk for suicide.

11. What is the rationale for a nurse to perform a full physical health assessment on a client admitted with a diagnosis of major depressive episode? 1. The attention during the assessment is beneficial in decreasing social isolation. 2. Depression can generate somatic symptoms that can mask actual physical disorders. 3. Physical health complications are likely to arise from antidepressant therapy. 4. Depressed clients avoid addressing physical health and ignore medical problems.

ANS: 2 Rationale: The nurse should determine that a client with a diagnosis of major depressive episode needs a full physical health assessment, because depression can generate somatic symptoms that can mask actual physical disorders.

13. A client with a history of cerebrovascular accident (CVA) is brought to an emergency department experiencing memory problems, confusion, and disorientation. Based on this client's assessment data, which diagnosis would the nurse expect the physician to assign? 1. Delirium due to adverse effects of cardiac medications 2. Vascular neurocognitive disorder 3. Altered thought processes 4. Alzheimer's disease

ANS: 2 Rationale: The nurse should expect that the client will be diagnosed with vascular NCD, which is caused by significant cerebrovascular disease. Vascular NCD often has an abrupt onset. Progression of this disease often occurs in a fluctuating pattern.

16. A client who has been taking fluvoxamine (Luvox) without significant improvement asks a nurse, "I heard about something called a monoamine oxidase inhibitor (MAOI). Can't my doctor add that to my medications?" Which is an appropriate nursing response? 1. "This combination of drugs can lead to delirium tremens." 2. "A combination of an MAOI and Luvox can lead to a life-threatening hypertensive crisis." 3. "That's a good idea. There have been good results with the combination of these two drugs." 4. "The only disadvantage would be the exorbitant cost of the MAOI.

ANS: 2 Rationale: The nurse should explain to the client that combining an MAOI and fluvoxamine, an SSRI, can lead to a life-threatening hypertensive crisis. Symptoms of hypertensive crisis include severe occipital and/or temporal pounding headaches, with occasional photophobia, sensations of choking, palpitations, and a feeling of "dread."

12. A client diagnosed with major NCD is exhibiting behavioral problems on a daily basis. At change of shift, the client's behavior escalates from pacing to screaming and flailing. Which action should be a nursing priority? 1. Consult the psychologist regarding behavior-modification techniques. 2. Medicate the client with prn antianxiety medications. 3. Assess environmental triggers and potential unmet needs. 4. Anticipate the behavior and restrain when pacing begins.

ANS: 2 Rationale: The priority nursing action is to first medicate the client to avoid injury to self or others. It is important to assess environmental triggers and potential unmet needs in order to address these problems in the future, but interventions to ensure safety must take priority. Because of the cognitive decline experienced in clients diagnosed with this disorder, communication skills and orientation may limit assessment and teaching interventions.

18. The severity of depressive symptoms in the postpartum period varies from a feeling of the "blues," to moderate depression, to psychotic depression or melancholia. Which disorder is correctly matched with its presenting symptoms? 1. Maternity blues (lack of concentration, agitation, guilt, and an abnormal attitude toward bodily functions) 2. Postpartum depression (irritability, loss of libido, sleep disturbances, expresses concern about inability to care for baby) 3. Postpartum melancholia (overprotection of infant, expresses concern about inability to care for baby, mysophobia) 4. Postpartum depressive psychosis (transient depressed mood, agitation, abnormal fear of child abduction, suicidal ideations)

ANS: 2 Rationale: The symptoms of the maternity blues include tearfulness, despondency, anxiety, and subjectively impaired concentration appearing in the early puerperium. Symptoms of postpartum depression are associated with fatigue, irritability, loss of appetite, sleep disturbances, loss of libido, and expressions of great concern about her inability to care for her baby. Both postpartum melancholia and postpartum depressive psychosis are characterized by a lack of interest in, or rejection of, the baby, or a morbid fear that the baby may be harmed. Other symptoms include depressed mood, agitation, indecision, lack of concentration, guilt, and an abnormal attitude toward bodily functions.

15. A client has been assigned an admission diagnosis of brief psychotic disorder. Which assessment information would alert the nurse to question this diagnosis? 1. The client has experienced impaired reality testing for a 24-hour period. 2. The client has experienced auditory hallucinations for the past 3 hours. 3. The client has experienced bizarre behavior for 1 day. 4. The client has experienced confusion for 3 weeks.

ANS: 2 This disorder is identified by the sudden onset of psychotic symptoms that may or may not be preceded by a severe psychosocial stressor. These symptoms last at least 1 day but less than 1 month. Page: 348 Feedback 1 Impaired reality testing for a 24-hour period is typical of brief psychotic disorder and would not cause the nurse to question this diagnosis. 2 This disorder is identified by the sudden onset of psychotic symptoms that may or may not be preceded by a severe psychosocial stressor. These symptoms last at least 1 day but less than 1 month. 3 Bizarre behavior for 1 day is typical of brief psychotic disorder and would not cause the nurse to question this diagnosis. 4 Confusion for 3 weeks is typical of brief psychotic disorder and would not cause the nurse to question this diagnosis.

23. An individual experiences sadness and melancholia in September continuing through November. Which of the following factors should a nurse identify as most likely to contribute to the etiology of these symptoms? (Select all that apply.) 1. Gender differences in social opportunities that occur with age 2. Drastic temperature and barometric pressure changes 3. A seasonal increase in social interactions 4. Variations in serotonergic functioning 5. Inaccessibility of resources for dealing with life stressors

ANS: 2, 3, 4 Rationale: The nurse should identify drastic temperature and barometric pressure changes, a seasonal increase in social interactions, and/or variations in serotonergic functioning as contributing to the etiology of the client's symptoms. A number of studies have examined seasonal patterns associated with mood disorders and have revealed two prevalent periods of seasonal involvement: spring (March, April, May) and fall (September, October, November).

10. A tearful, anxious patient at the outpatient clinic reports, "I should be dead." The initial task of the nurse conducting the assessment interview is to: a. assess the lethality of a suicide plan. b. encourage expression of anger. c. establish a rapport with the patient. d. determine risk factors for suicide.

ANS: C Establishing rapport will allow the nurse to obtain relevant assessment data such as the presence of a suicide plan, the lethality of a suicide plan, and the presence of risk factors for suicide.

16. A nurse is assessing a client diagnosed with substance induced psychotic disorder (SIPD). What would differentiate this client's symptoms from the symptoms of a client diagnosed with brief psychotic disorder (BPD)? 1. Clients diagnosed with SIPD experience delusions, whereas clients diagnosed with BPD do not. 2. Clients diagnosed with BPD experience hallucinations, whereas clients diagnosed with SIPD do not. 3. Catatonic features may be associated with SIPD, whereas BPD has no catatonic features. 4. Catatonic features may be associated with BPD, whereas SIPD has no catatonic features.

ANS: 3 Page: 348 Feedback 3 Catatonic features may be associated with SIPD, whereas BPD has no catatonic features.

13. A nurse admits an older client with memory loss, confused thinking, and apathy. A psychiatrist suspects a depressive disorder. What is the rationale for performing a mini-mental status exam? 1. To rule out bipolar disorder 2. To rule out schizophrenia 3. To rule out neurocognitive disorder 4. To rule out personality disorder

ANS: 3 Rationale: A mini-mental status exam should be performed to rule out neurocognitive disorder. The client may be experiencing reversible dementia, which can occur as a result of depression.

5. A client is diagnosed in stage seven of AD. To address the client's symptoms, which nursing intervention should take priority? 1. Improve cognitive status by encouraging involvement in social activities. 2. Decrease social isolation by providing group therapies. 3. Promote dignity by providing comfort, safety, and self-care measures. 4. Facilitate communication by providing assistive devices.

ANS: 3 Rationale: The most appropriate intervention in the seventh stage of AD is to promote the client's dignity by providing comfort, safety, and self-care measures. Stage is characterized by severe cognitive decline in which the client is unable to recognize family members and is most commonly bedfast and aphasic.

9. A client diagnosed with AD exhibits progressive memory loss, diminished cognitive functioning, and verbal aggression upon experiencing frustration. Which nursing intervention is most appropriate? 1. Organize a group activity to present reality. 2. Minimize environmental lighting. 3. Schedule structured daily routines. 4. Explain the consequences for aggressive behaviors.

ANS: 3 Rationale: The most appropriate nursing intervention for this client is to schedule structured daily routines. A structured routine will reduce frustration and thereby reduce verbal aggression.

. A nurse administers 100% oxygen to a client during and after electroconvulsive therapy treatment (ECT). What is the rationale for this procedure? 1. To prevent increased intracranial pressure resulting from anoxia. 2. To prevent decreased blood pressure, pulse, and respiration owing to electrical stimulation. 3. To prevent anoxia resulting from medication-induced paralysis of respiratory muscles. 4. To prevent blocked airway, resulting from seizure activity.

ANS: 3 Rationale: The nurse administers 100% oxygen during and after ECT to prevent anoxia resulting from medication-induced paralysis of respiratory muscles.

10. A depressed client reports to a nurse a history of divorce, job loss, family estrangement, and cocaine abuse. Which theoretical principle best explains the etiology of this client's depressive symptoms? 1. According to psychoanalytic theory, depression is a result of negative perceptions. 2. According to object-loss theory, depression is a result of overprotection. 3. According to learning theory, depression is a result of repeated failures. 4. According to cognitive theory, depression is a result of anger turned inward.

ANS: 3 Rationale: The nurse should assess that, according to learning theory, this client's depressive symptoms may have resulted from repeated failures. The learning theory is a model of "learned helplessness" in which multiple life failures cause the client to abandon future attempts to succeed.

8. A client with a history of suicide attempts has been taking fluoxetine (Prozac) for one month. The client suddenly presents with a bright affect, rates mood at 9 out of 10, and is much more communicative. Which action should be the nurse's priority at this time? 1. Give the client off-unit privileges as positive reinforcement. 2. Encourage the client to share mood improvement in group. 3. Increase the level of this client's suicide precautions. 4. Request that the psychiatrist reevaluate the current medication protocol.

ANS: 3 Rationale: The nurse should be aware that a sudden increase in mood rating and change in affect could indicate that the client is at risk for suicide. Suicide risk may occur early during treatment with antidepressants. The return of energy may bring about an increased ability to act out self-destructive behavior.

8. Which symptom should a nurse identify that differentiates clients diagnosed with NCDs from clients diagnosed with mood disorders? 1. Altered sleep 2. Altered concentration 3. Impaired memory 4. Impaired psychomotor activity

ANS: 3 Rationale: The nurse should identify that impaired memory is a symptom that occurs in NCD and not in mood disorders. Neurocognitive disorder is classified in the DSM-5 as either mild or major, with the distinction primarily being one of severity of symptomatology.

10. After one week of continuous mental confusion, an older African American client is admitted with a preliminary diagnosis of AD. What should cause the nurse to question this diagnosis? 1. AD does not typically occur in African American clients. 2. The symptoms presented are more indicative of Parkinsonism. 3. AD does not develop suddenly. 4. There has been no T3- or T4-level evaluation ordered.

ANS: 3 Rationale: The nurse should recognize that AD does not develop suddenly and should question this diagnosis. The onset of AD symptoms is slow and insidious. The disease is generally progressive and deteriorating.

6. A client diagnosed with major depressive episode hears voices commanding self-harm. Which should be the nurse's priority intervention at this time? 1. Obtaining an order for locked seclusion until client is no longer suicidal. 2. Conducting 15-minute checks to ensure safety. 3. Placing the client on one-to-one observation while continuing to monitor suicidal ideations. 4. Encouraging client to express feelings related to suicide.

ANS: 3 Rationale: The nurse's priority intervention when a depressed client hears voices commanding self-harm is to place the client on one-to-one observation while continuing to monitor suicidal ideations. By providing one-to-one observation, the nurse will be able to interrupt any attempts at suicide.

A patient with severe dementia no longer recognizes family members and becomes anxious and agitated when they attempt reorientation. Which alternative could the nurse suggest to the family members? A: Wear large name tags B: Focus interaction on familiar topics C: Frequently repeat the reorientation strategies D: Place large clocks and calendars strategically

B: Focus interaction on familiar topics

1. A nurse discovers a client's suicide note that details the time, place, and means to commit suicide. What should be the priority nursing action, and why? 1. Administer lorazepam (Ativan) prn, because the client is angry about plan exposure. 2. Establish room restrictions, because the client's threat is an attempt to manipulate the staff. 3. Place client on one-to-one suicide precautions, because specific plans likely lead to attempts. 4. Call an emergency treatment team meeting, because the client's threat must be addressed.

ANS: 3 Rationale: The priority nursing action should be to place this client on one-to-one suicide precautions, because the more specific the plan, the more likely the client will attempt suicide.

15. A client diagnosed with NCD is disoriented and ataxic and wanders. Which is the priority nursing diagnosis? 1. Disturbed thought processes 2. Self-care deficit 3. Risk for injury 4. Altered health-care maintenance

ANS: 3 Rationale: The priority nursing diagnosis for this client is risk for injury. The client who is ataxic suffers from motor coordination deficits and is at an increased risk for falls. Clients that wander are at a higher risk for injury.

16. Which statement accurately differentiates mild NCD from major NCD? 1. Major NCD involves disorientation that develops suddenly, whereas mild NCD develops more slowly. 2. Major NCD involves impairment of abstract thinking and judgment, whereas mild NCD does not. 3. Major NCD criteria requires substantial cognitive decline from a previous level of performance, and mild NCD requires modest decline. 4. Major NCD criteria requires decline from a previous level of performance in three of the listed domains, and mild NCD requires only one.

ANS: 3 Rationale: The progression of the disorder is not a criterion for determining the severity of an NCD. Abstract thinking and judgment can be affected in both mild NCD and major NCD. Major NCD criteria requires substantial cognitive decline, and mild NCD requires modest decline. Both major and mild NCD classifications require decline from a previous level of performance in only one of the listed domains.

19. A staff nurse is counseling a depressed client. The nurse determines that the client is using the cognitive distortion of "automatic thoughts." Which client statement is evidence of the "automatic thought" of discounting positives? 1. "It's all my fault for trusting him." 2. "I don't play games. I never win." 3. "She never visits because she thinks I don't care." 4. "I don't have a green thumb. Any old fool can grow a rose."

ANS: 4 Rationale: Examples of automatic thoughts in depression include: Personalizing: "I'm the only one who failed." All or nothing: "I'm a complete failure." Mind reading: "He thinks I'm foolish." Discounting positives: "The other questions were so easy. Any dummy could have gotten them right."

12. A nurse is planning care for a 13 -year-old who is experiencing depression. Which medication is approved by the Food and Drug Administration (FDA) for the treatment of depression in adolescents? 1. Paroxetine (Paxil) 2. Sertraline (Zoloft) 3. Citalopram (Celexa) 4. Escitalopram (Lexipro)

ANS: 4 Rationale: Fluoxetine (Prozac) has been approved by the FDA to treat depression in children and adolescents, and escitalopram was approved in 2009 for treatment of depression in adolescents aged 12 to 17 years. All antidepressants carry an FDA warning for increased risk of suicide in children and adolescents.

7. A nurse assesses a client suspected of having the diagnosis of major depressive episode. Which client symptom would rule out this diagnosis? 1. The client is disheveled and malodorous. 2. The client refuses to interact with others and isolates self in room. 3. The client is unable to feel any pleasure. 4. The client has maxed-out charge cards and exhibits promiscuous behaviors.

ANS: 4 Rationale: The nurse should assess that a client who has maxed-out credit cards and exhibits promiscuous behavior is exhibiting signs of mania. The DSM-5 criteria state that there must never have been a manic episode or a hypomanic episode to meet the criteria for the diagnosis of major depressive episode.

14. An older client has recently moved to a nursing home. The client has trouble concentrating and socially isolates. A physician believes the client would benefit from medication therapy. Which medication should the nurse expect the physician to prescribe? 1. Haloperidol (Haldol) 2. Donepezil (Aricept) 3. Diazepam (Valium) 4. Sertraline (Zoloft)

ANS: 4 Rationale: The nurse should expect the physician to prescribe sertraline to improve the client's social functioning and concentration levels. Sertraline is an selective serotonin reuptake inhibitor (SSRI) antidepressant. Depression is the most common mental illness in older adults and is often misdiagnosed as a neurocognitive disorder.

3. A client diagnosed with vascular neurocognitive disorder (NCD) is discharged to home under the care of his wife. Which information should cause the nurse to question the client's safety? 1. His wife works from home in telecommunication. 2. The client has worked the nightshift his entire career. 3. His wife has minimal family support. 4. The client smokes one pack of cigarettes per day.

ANS: 4 Rationale: The nurse should question the client's safety at home if the client smokes cigarettes. Vascular NCD is a clinical syndrome of NCD due to significant cerebrovascular disease. The cause of vascular NCD is related to an interruption of blood flow to the brain. Hypertension is a significant factor in the etiology.

4. A client diagnosed with AD can no longer ambulate, does not recognize family members, and communicates with agitated behaviors and incoherent verbalizations. The nurse recognizes these symptoms as indicative of which stage of the illness? 1. Stage 4: Mild-to-Moderate Cognitive Decline 2. Stage 5. Moderate Cognitive Decline 3. Stage 6. Moderate-to-Severe Cognitive Decline 4. Stage 7. Severe Cognitive Decline

ANS: 4 Rationale: The nurse should recognize that a client exhibiting these symptoms is in the severe cognitive decline, seventh stage, of AD.

15. An older client has recently been prescribed sertraline (Zoloft). The client's spouse is taking paroxetine (Paxil). A nurse assesses that the client is experiencing restlessness, tachycardia, diaphoresis, and tremors. Which complication should a nurse suspect, and why? 1. Neuroleptic malignant syndrome; caused by ingestion of two different seratonin reuptake inhibitors (SSRIs) 2. Neuroleptic malignant syndrome; caused by ingestion of an SSRI and a monoamine oxidase inhibitor (MAOI) 3. Serotonin syndrome; possibly caused by ingestion of an SSRI and an MAOI 4. Serotonin syndrome; possibly caused by ingestion of two different SSRIs

ANS: 4 Rationale: The nurse should suspect that the client is suffering from serotonin syndrome; possibly caused by ingesting two different SSRI's (sertraline and paroxetine). Symptoms of serotonin syndrome include confusion, agitation, tachycardia, hypertension, nausea, abdominal pain, myoclonus, muscle rigidity, fever, sweating, and tremor.

2. In planning care for a suicidal client, which correctly written outcome should be a nurse's first priority? 1. The client will not physically harm self. 2. The client will express hope for the future by day three. 3. The client will establish a trusting relationship with the nurse. 4. The client will remain safe during hospital stay.

ANS: 4 Rationale: The nurse's first priority should be that the client will remain safe during the hospital stay. Client safety should always be the nurse's first priority. Outcomes should be client-centered, specific, realistic, measureable, and must also include a time frame.

11. A client diagnosed with AD has impairments of memory and judgment and is incapable of performing activities of daily living. Which nursing intervention should take priority? 1. Present evidence of objective reality to improve cognition. 2. Design a bulletin board to represent the current season. 3. Label the client's room with name and number. 4. Assist with bathing and toileting.

ANS: 4 Rationale: The priority nursing intervention for this client is to assist with bathing and toileting. A client who is incapable of performing activities of daily living requires assistance in these areas to ensure health and safety.

6. Which is the reason for the proliferation of the diagnosis of NCDs? 1. Increased numbers of neurotransmitters has been implicated in the proliferation of NCD. 2. Similar symptoms of NCD and depression lead to misdiagnoses, increasing numbers of NCD. 3. Societal stress contributes to the increase in this diagnosis. 4. More people now survive into the high-risk period for neurocognitive disorders.

ANS: 4 Rationale: The proliferation of NCD has occurred because more people now survive into the high-risk period for neurocognitive disorder, which is middle age and beyond..

13. Which statement provides the best rationale for why a nurse should closely monitor a severely depressed patient during antidepressant medication therapy? a. As depression lifts, physical energy becomes available to carry out suicide. b. Suicide may be precipitated by a variety of internal and external events. c. Suicidal patients have difficulty using social supports. d. Suicide is an impulsive act.

ANS: A Antidepressant medication has the objective of relieving depression. The risk for suicide is greater as the depression lifts, primarily because the patient has more physical energy at a time when he or she may still have suicidal ideation. The other options have little to do with nursing interventions relating to antidepressant medication therapy.

20. The feeling experienced by a patient that should be assessed by the nurse as most predictive of elevated suicide risk is: a. hopelessness. b. sadness. c. elation. d. anger.

ANS: A Of the feelings listed, hopelessness is most closely associated with increased suicide risk. Depression, aggression, impulsivity, and shame are other feelings noted as risk factors for suicide.

15. A depressed patient says, "Nothing matters anymore." What is the most appropriate response by the nurse? a. "Are you having thoughts of suicide?" b. "I am not sure I understand what you are trying to say." c. "Try to stay hopeful. Things have a way of working out." d. "Tell me more about what interested you before you began feeling depressed."

ANS: A The nurse must make overt what is covert; that is, the possibility of suicide must be openly addressed. Often, patients feel relieved to be able to talk about suicidal ideation.

6. A college student who attempted suicide by overdose is hospitalized. When the parents are contacted, they respond, "There must be a mistake. This could not have happened. We've given our child everything." The parents' reaction reflects: a. denial. b. anger. c. anxiety. d. rescue feelings.

ANS: A The parents' statements indicate denial. Denial or minimization of suicidal ideation or attempts is a defense against uncomfortable feelings. Family members are often unable to acknowledge suicidal ideation in someone close to them. The feelings suggested in the distractors are not clearly described in the scenario.

8. An adult attempts suicide after declaring bankruptcy. The patient is hospitalized and takes an antidepressant medication for five days. The patient is now more talkative and shows increased energy. Select the highest priority nursing intervention. a. Supervise the patient 24 hours a day. b. Begin discharge planning for the patient. c. Refer the patient to art and music therapists. d. Consider the discontinuation of suicide precautions.

ANS: A The patient now has more energy and may have decided on suicide, especially considering the history of the prior suicide attempt. The patient is still a suicide risk; therefore, continuous supervision is indicated.

21. Four individuals have given information about their suicide plans. Which plan evidences the highest suicide risk? a. Jumping from a 100-foot-high railroad bridge located in a deserted area late at night b. Turning on the oven and letting gas escape into the apartment during the night c. Cutting the wrists in the bathroom while the spouse reads in the next room d. Overdosing on aspirin with codeine while the spouse is out with friends

ANS: A This is a highly lethal method with little opportunity for rescue. The other options are lower lethality methods with higher rescue potential.

28. The parents of identical twins ask a nurse for advice. One twin committed suicide a month ago. Now the parents are concerned that the other twin may also have suicidal tendencies. Which comment by the nurse is accurate? a. "Genetics are associated with suicide risk. Monitoring and support are important." b. "Apathy underlies suicide. Instilling motivation is the key to health maintenance." c. "Your child is unlikely to act out suicide when identifying with a suicide victim." d. "Fraternal twins are at higher risk for suicide than identical twins."

ANS: A Twin studies suggest the presence of genetic factors in suicide; however, separating genetic predisposition to suicide from predisposition to depression or alcoholism is difficult. Primary interventions can be helpful in promoting and maintaining health and possibly counteracting the genetic load. The incorrect options are untrue statements or oversimplifications.

2. A college student failed two tests. Afterward, the student cried for hours and then tried to telephone a parent but got no answer. The student then gave several expensive sweaters to a roommate. Which behavior provides the strongest clue of an impending suicide attempt? a. Calling parents b. Excessive crying c. Giving away sweaters d. Staying alone in a dorm room

ANS: C Giving away prized possessions may signal that the individual thinks he or she will have no further need for the items, such as when a suicide plan has been formulated. Calling parents and crying do not provide clues to suicide, in and of themselves. Remaining in the dormitory would be an expected behavior because the student has nowhere else to go.

27. A new nurse says to a peer, "My newest patient is diagnosed with schizophrenia. At least I won't have to worry about suicide risk." Which response by the peer would be most helpful? a. "Let's reconsider your plan. Suicide risk is high in patients diagnosed with schizophrenia." b. "Suicide is a risk for any patient diagnosed with schizophrenia who uses alcohol or drugs." c. "Patients diagnosed with schizophrenia are usually too disorganized to attempt suicide." d. "Visual hallucinations often prompt suicide among patients diagnosed with schizophrenia."

ANS: A Up to 10% of patients diagnosed with schizophrenia die from suicide, usually related to depressive symptoms occurring in the early years of the illness. Depressive symptoms are related to suicide among patients diagnosed with schizophrenia. Patients diagnosed with schizophrenia usually have auditory, not visual, hallucinations. Although the use of drugs and alcohol compounds the risk for suicide, it is independent of schizophrenia.

2. A patient with suicidal impulses is on the highest level of suicide precautions. Which measures should the nurse incorporate into the patient's plan of care? Select all that apply. a. Allow no glass or metal on meal trays. b. Remove all potentially harmful objects from the patient's possession. c. Maintain arm's length, one-on-one nursing observation around the clock. d. Check the patient's whereabouts every hour. Make verbal contact at least three times each shift. e. Check the patient's whereabouts every 15 minutes, and make frequent verbal contacts. f. Keep the patient within visual range while he or she is awake. Check every 15 to 30 minutes while the patient is sleeping.

ANS: A, B, C One-on-one observation is necessary for anyone who has limited control over suicidal impulses. Plastic dishes on trays and the removal of potentially harmful objects from the patient's possession are measures included in any level of suicide precautions. The remaining options are used in less stringent levels of suicide precautions.

3. A nurse assesses five newly hospitalized patients. Which patients have the highest suicide risk? Select all that apply. a. 82-year-old white man b. 17-year-old white female adolescent c. 39-year-old African-American man d. 29-year-old African-American woman e. 22-year-old man with traumatic brain injury

ANS: A, B, E Whites have suicide rates almost twice those of nonwhites, and the rate is particularly high for older adult men, adolescents, and young adults. Other high risk groups include young African-American men, Native-American men, older Asian Americans, and persons with traumatic brain injury.

24. A staff nurse tells another nurse, "I evaluated a new patient using the SAD PERSONS scale and got a score of 10. I'm wondering if I should send the patient home." Select the best reply by the second nurse. a. "That action would seem appropriate." b. "A score over 8 requires immediate hospitalization." c. "I think you should strongly consider hospitalization for this patient." d. "Give the patient a follow-up appointment. Hospitalization may be needed soon."

ANS: B A SAD PERSONS scale score of 0 to 5 suggests home care with follow-up. A score of 6 to 8 requires psychiatric consultation. A score over 8 calls for hospitalization.

25. A patient recently hospitalized for two weeks committed suicide during the night. Which initial measure will be most helpful for staff members and other patients regarding this event? a. Request the public information officer to make an announcement to the local media. b. Hold a staff meeting to express feelings and plan the care for other patients. c. Ask the patient's roommate not to discuss the event with other patients. d. Quickly discharge as many patients as possible to prevent panic.

ANS: B Interventions should be aimed at helping the staff and patients come to terms with the loss and to grow because of the incident. Then, a community meeting should be scheduled to allow other patients to express their feelings and request help. Staff members should be prepared to provide additional support and reassurance to patients and should seek opportunities for peer support. The incorrect options will not control information or may result in unsafe care.

19. Which statement by a patient during an assessment interview should alert the nurse to the patient's need for immediate, active intervention? a. "I am mixed up, but I know I need help." b. "I have no one for help or support." c. "It is worse when you are a person of color." d. "I tried to get attention before I shot myself."

ANS: B Lack of social support and social isolation increase the suicide risk. The willingness to seek help lowers the risk. Being a person of color does not suggest a higher risk; more whites commit suicide than do individuals of other racial groups. Attention seeking is not correlated with a higher risk of suicide.

1. Which changes in brain biochemical function is most associated with suicidal behavior? a. Dopamine excess b. Serotonin deficiency c. Acetylcholine excess d. Gamma-aminobutyric acid deficiency

ANS: B Research suggests that low levels of serotonin may play a role in the decision to commit suicide. The other neurotransmitter alterations have not been implicated in suicidal crises.

12. Which intervention should a nurse recommend for the distressed family and friends of someone who has committed suicide? a. Participating in reminiscence therapy b. Attending a self-help group for survivors c. Contracting for two sessions of group therapy d. Completing a psychological postmortem assessment

ANS: B Survivors need outlets for their feelings about the loss and the deceased person. Self-help groups provide peer support while survivors work through feelings of loss, anger, and guilt. Psychological postmortem assessment would not provide the support necessary to work through feelings of loss associated with the suicide of a family member. Reminiscence therapy is not geared to loss resolution. Contracting for two sessions of group therapy would probably not provide sufficient time to work through the issues associated with a death by suicide.

7. An adolescent tells the school nurse, "My friend threatened to take an overdose of pills." The nurse talks to the friend who verbalized the suicide threat. The most critical question for the nurse to ask would be: a. "Why do you want to kill yourself?" b. "Do you have access to medications?" c. "Have you been taking drugs and alcohol?" d. "Did something happen with your parents?"

ANS: B The nurse must assess the patient's access to the means to carry out the plan and, if there is access, alert the parents to remove them from the home. The other questions may be important to ask but are not the most critical.

17. When assessing a patient's plan for suicide, what aspect has priority? a. Patient's financial and educational status b. Patient's insight into suicidal motivation c. Availability of means and lethality of method d. Quality and availability of patient's social support

ANS: C If a person has definite plans that include choosing a method of suicide readily available, and if the method is one that is lethal (i.e., will cause the person to die with little probability for intervention), the suicide risk is considered high. These areas provide a better indication of risk than the areas mentioned in the other options.

3. A nurse uses the SAD PERSONS scale to interview a patient. This tool provides data relevant to: a. current stress level. b. mood disturbance. c. suicide potential. d. level of anxiety.

ANS: C The SAD PERSONS tool evaluates 10 major risk factors in suicide potential: sex, age, depression, previous attempt, ethanol use, rational thinking loss, social supports lacking, organized plan, no spouse, and sickness. The tool does not have appropriate categories to provide information on the other options listed.

9. A nurse and patient construct a no-suicide contract. Select the preferable wording for the contract. a. "I will not try to harm myself during the next 24 hours." b. "I will not make a suicide attempt while I am hospitalized." c. "For the next 24 hours, I will not kill or harm myself in any way." d. "I will not kill myself until I call my primary nurse or a member of the staff."

ANS: C The correct answer leaves no loopholes. The wording about not harming oneself and not making an attempt leaves loopholes or can be ignored by the patient who thinks, "I am not going to harm myself, I am going to kill myself," or "I am not going to attempt suicide, I am going to commit suicide." A patient may call a therapist and leave the telephone to carry out the suicidal plan.

4. A person intentionally overdoses on antidepressant drugs. Which nursing diagnosis has the highest priority? a. Powerlessness b. Social isolation c. Risk for suicide d. Ineffective management of the therapeutic regimen

ANS: C This diagnosis is the only one with life-or-death ramifications and is therefore higher in priority than the other options.

14. A nurse assesses a patient who reports a 3-week history of depression and crying spells. The patient says, "My business is bankrupt, and I was served with divorce papers." Which subsequent statement by the patient alerts the nurse to a concealed suicidal message? a. "I wish I were dead." b. "Life is not worth living." c. "I have a plan that will fix everything." d. "My family will be better off without me."

ANS: C Verbal clues to suicide may be overt or covert. The incorrect options are overt references to suicide. The correct option is more veiled. It alludes to the patient's suicide as being a way to "fix everything" but does not say it outright.

1. A college student failed two examinations. The student cried for hours and then tried to call a parent but got no answer. The student then suspended access to his social networking web site. Which suicide risk factors are present? Select all that apply. a. History of earlier suicide attempt b. Co-occurring medical illness c. Recent stressful life event d. Self-imposed isolation e. Shame or humiliation

ANS: C, D, E Failing examinations in the academic major constitutes a recent stressful life event. Shame and humiliation related to the failure can be hypothesized. The inability to contact parents can be seen as a recent lack of social support, as can the roommate's absence from the dormitory. Terminating access to one's social networking site represents self-imposed isolation. This scenario does not provide data regarding a history of an earlier suicide attempt, a family history of suicide, or of co-occurring medical illness.

4. A nurse assesses the health status of soldiers returning from Afghanistan. Screening for which health problems will be a priority? Select all that apply. a. Schizophrenia b. Eating disorder c. Traumatic brain injury d. Oppositional defiant disorder e. Post-traumatic stress disorder

ANS: C, E Traumatic brain injury and post-traumatic stress disorder each occur in approximately 20% of soldiers returning from Afghanistan. Some soldiers have both problems. The incidence of disorders identified in the distractors would be expected to parallel the general population.

22. Which individual in the emergency department should be considered at the highest risk for completing suicide? a. An adolescent Asian-American girl with superior athletic and academic skills who has asthma b. A 38-year-old single African-American female church member with fibrocystic breast disease c. A 60-year-old married Hispanic man with 12 grandchildren who has type 2 diabetes d. A 79-year-old single white man with cancer of the prostate gland

ANS: D High-risk factors include being an older adult, single, and male and having a co-occurring medical illness. Cancer is one of the somatic conditions associated with increased suicide risk. Protective factors for African-American women and Hispanic individuals include strong religious and family ties. Asian Americans have a suicide rate that increases with age.

18. Which understanding about individuals who attempt suicide will help a nurse plan the care for a suicidal patient? Every suicidal person should be considered: a. mentally ill. b. intent on dying. c. cognitively impaired. d. experiencing hopelessness.

ANS: D Hopelessness is the characteristic common among people who attempt suicide. The incorrect options reflect myths about suicide. Not all who attempt suicide are intent on dying. Not all are mentally ill or cognitively impaired.

5. A person attempts suicide by overdose, is treated in the emergency department, and then hospitalized. What is the best initial outcome? The patient will: a. verbalize a will to live by the end of the second hospital day. b. describe two new coping mechanisms by the end of the third hospital day. c. accurately delineate personal strengths by the end of first week of hospitalization. d. exercise suicide self-restraint by refraining from gestures or attempts to kill self for 24 hours.

ANS: D Suicide self-restraint relates most directly to the priority problem of risk for self-directed violence. The other outcomes are related to hope, coping, and self-esteem.

16. A nurse counsels a patient with recent suicidal ideation. Which is the nurse's most therapeutic comment? a. "Let's make a list of all your problems and think of solutions for each one." b. "I'm happy you're taking control of your problems and trying to find solutions." c. "When you have bad feelings, try to focus on positive experiences from your life." d. "Let's consider which problems are most important and which are less important."

ANS: D The nurse helps the patient develop effective coping skills. He or she assists the patient to reduce the overwhelming effects of problems by prioritizing them. The incorrect options continue to present overwhelming approaches to problem solving.

23. A nurse answers a suicide crisis line. A caller says, "I live alone in a home several miles from my nearest neighbors. I have been considering suicide for 2 months. I have had several drinks and now my gun is loaded. I'm going to shoot myself in the heart." How would the nurse assess the lethality of this plan? a. No risk b. Low level c. Moderate level d. High level

ANS: D The patient has a highly detailed plan, a highly lethal method, the means to carry it out, lowered impulse control because of alcohol ingestion, and a low potential for rescue.

11. Select the most helpful response for a nurse to make when a patient being treated as an outpatient states, "I am considering suicide." a. "I'm glad you shared this. Please do not worry. We will handle it together." b. "I think you should admit yourself to the hospital to get help." c. "We need to talk about the good things you have to live for." d. "Bringing this up is a very positive action on your part."

ANS: D This response gives the patient reinforcement and validation for making a positive response rather than acting out the suicidal impulse. It gives neither advice nor false reassurance, and it does not imply stereotypes such as, "You have a lot to live for." It uses the patient's ambivalence and sets the stage for more realistic problem-solving strategies.

26. A severely depressed patient who has been on suicide precautions tells the nurse, "I am feeling a lot better, so you can stop watching me. I have taken too much of your time already." Which is the nurse's best response? a. "I wonder what this sudden change is all about. Please tell me more." b. "I am glad you are feeling better. The team will consider your request." c. "You should not try to direct your care. Leave that to the treatment team." d. "Because we are concerned about your safety, we will continue with our plan."

ANS: D When a patient seeks to have precautions lifted by professing to feel better, the patient may be seeking greater freedom in which to attempt suicide. Changing the treatment plan requires careful evaluation of outcome indicators by the staff. The incorrect options will not cause the patient to admit to a suicidal plan, do not convey concern for the patient, or suggest that the patient is not a partner in the care process.

. ___________________________ are false sensory perceptions not associated with real external stimuli and may involve any of the five senses.

ANS: Hallucinations Rationale: Hallucinations are false sensory perceptions not associated with real external stimuli and may involve any of the five senses. Types of hallucinations include auditory, visual, tactile, gustatory, and olfactory.

27. ___________________________ is a pervasive and sustained emotion that may have a major influence on a person's perception of the world.

ANS: Mood Rationale: Mood is a pervasive and sustained emotion that may have a major influence on a person's perception of the world. Examples of mood include depression, joy, elation, anger, and anxiety. Affect is described as the emotional reaction associated with an experience.

. ___________________________ disorder is manifested by schizophrenic behaviors, with a strong element of symptomatology associated with the mood disorders (depression or mania).

ANS: Schizoaffective Rationale: Schizoaffective disorder is manifested by schizophrenic behaviors, with a strong element of symptomatology associated with the mood disorders (depression or mania). The decisive factor in the diagnosis of schizoaffective disorder is the presence of hallucinations and/or delusions that occur for at least 2 weeks in the absence of a major mood episode.

26. Order the spectrum of schizophrenic and other psychotic disorders as described by the DSM-5 on a gradient of psychopathology from least to most severe. ________ Delusional disorder ________ Schizotypal personality disorder ________ Schizophrenia ________ Brief psychotic disorder ________ Psychotic disorder associated with another medical condition ________ Catatonic disorder associated with another medical condition ________ Schizoaffective disorder ________ Schizophreniform disorder ________ Substance-induced psychotic disorder

ANS: The correct order is 2, 1, 9, 3, 5, 6, 8, 7, 4 Rationale: A spectrum of schizophrenic and other psychotic disorders has been identified in the DSM-5. These include (on a gradient of psychopathology from least to most severe): schizotypal personality disorder, delusional disorder, brief psychotic disorder, substance-induced psychotic disorder, psychotic disorder associated with another medical condition, catatonic disorder associated with another medical condition, schizophreniform disorder, schizoaffective disorder, and schizophrenia. 1. Schizotypal personality disorder 2. Delusional disorder 3. Brief psychotic disorder 4. Substance-induced psychotic disorder 5. Psychotic disorder associated with another medical condition 6. Catatonic disorder associated with another medical condition 7. Schizophreniform disorder 8. Schizoaffective disorder 9. Schizophrenia

26. Order the depressive disorders and their predominant affective symptoms according to level of severity. ________ Dysthymic disorder (pessimistic outlook, low self-esteem) ________ Grief (feelings of anger, anxiety, guilt, helplessness) ________ Major depressive episode (despair, worthlessness, flat affect, apathy, anhedonia) ________ Transient depression (sadness, dejection, feeling downhearted, having "the blues")

ANS: The correct order is 3, 2, 4, 1 Rationale: Symptoms of transient depression are not necessarily dysfunctional. Affective symptoms include sadness, dejection, feeling downhearted, having the "blues." Symptoms at the mild level of depression are identified by those associated with uncomplicated grieving. Affective symptoms include denial of feelings, anger, anxiety, guilt, helplessness, hopelessness, sadness, despondency. Dysthymic disorder, which is an example of moderate depression, represents a more problematic disturbance. Affective symptoms include feelings of sadness, dejection, helplessness, powerlessness, hopelessness; gloomy and pessimistic outlook; low self-esteem; difficulty experiencing pleasure in activities. Severe depression is characterized by an intensification of the symptoms described for moderate depression. Examples of severe depression include major depressive episode. Affective symptoms include feelings of total despair, hopelessness, and worthlessness; flat (unchanging) affect, appearing devoid of emotional tone; prevalent feelings of nothingness and emptiness; apathy; loneliness; sadness; inability to feel pleasure. 1. Transient depression (sadness, dejection, feeling downhearted, having "the blues") 2. Grief (feelings of anger, anxiety, guilt, helplessness) 3. Dysthymic disorder (pessimistic outlook, low self-esteem) 4. Major depressive episode (despair, worthlessness, flat affect, apathy, anhedonia)

Which nursing behavior will enhance the establishment of a trusting relationship with a client diagnosed with schizophrenia? A. Establishing personal contact with family members. B. Being reliable, honest, and consistent during interactions. C. Sharing limited personal information. D. Sitting close to the client to establish rapport.

B (The nurse can enhance the establishment of a trusting relationship with a client diagnosed with schizophrenia by being reliable, honest, and consistent during interactions. The nurse should also convey acceptance of the client's needs and maintain a calm attitude when dealing with agitated behavior.)

During an admission assessment, a nurse asks a client diagnosed with schizophrenia, "Have you ever felt that certain objects or persons have control over your behavior?" The nurse is assessing for which type of thought disruption? A. Delusions of persecution B. Delusions of influence C. Delusions of reference D. Delusions of grandeur

B (The nurse is assessing the client for delusions of influence when asking if the client has ever felt that objects or persons have control of the client's behavior. Delusions of control or influence are manifested when the client believes that his or her behavior is being influenced. An example would be if a client believes that a hearing aid receives transmissions that control personal thoughts and behaviors.)

A client diagnosed with schizophrenia tells a nurse, "The 'Shopatouliens' took my shoes out of my room last night." Which is an appropriate charting entry to describe this client's statement? A. "The client is experiencing command hallucinations." B. "The client is expressing a neologism." C. "The client is experiencing a paranoid delusion." D. "The client is verbalizing a word salad."

B (The nurse should describe the client's statement as experiencing a neologism. A neologism is when a client invents a new word that is meaningless to others but may have symbolic meaning to the client. Word salad refers to a group of words that are put together randomly.)

A client has been recently admitted to an inpatient psychiatric unit. Which intervention should the nurse plan to use to reduce the client's focus on delusional thinking? A. Present evidence that supports the reality of the situation B. Focus on feelings suggested by the delusion C. Address the delusion with logical explanations D. Explore reasons why the client has the delusion

B (The nurse should focus on the client's feelings rather than attempt to change the client's delusional thinking by the use of evidence or logical explanations. Delusional thinking is usually fixed, and clients will continue to have the belief in spite of obvious proof that the belief is false or irrational.)

A paranoid client presents with bizarre behaviors, neologisms, and thought insertion. Which nursing action should be prioritized to maintain this client's safety? A. Assess for medication noncompliance B. Note escalating behaviors and intervene immediately C. Interpret attempts at communication D. Assess triggers for bizarre, inappropriate behaviors

B (The nurse should note escalating behaviors and intervene immediately to maintain this client's safety. Early intervention may prevent an aggressive response and keep the client and others safe.)

A nurse is caring for a client who is experiencing a flat affect, paranoid delusions, anhedonia, anergia, neologisms, and echolalia. Which statement correctly differentiates the client's positive and negative symptoms of schizophrenia? A. Paranoid delusions, anhedonia, and anergia are positive symptoms of schizophrenia. B. Paranoid delusions, neologisms, and echolalia are positive symptoms of schizophrenia. C. Paranoid delusions, anergia, and echolalia are negative symptoms of schizophrenia. D. Paranoid delusions, flat affect, and anhedonia are negative symptoms of schizophrenia.

B (The nurse should recognize that positive symptoms of schizophrenia include paranoid delusions, neologisms, and echolalia. The negative symptoms of schizophrenia include flat affect, anhedonia, and anergia. Positive symptoms reflect an excess or distortion of normal functions. Negative symptoms reflect a decrease or loss of normal functions.)

A college student has quit attending classes, isolates self due to hearing voices, and yells accusations at fellow students. Based on this information, which nursing diagnosis should the nurse prioritize? A. Altered thought processes R/T hearing voices AEB increased anxiety B. Risk for other-directed violence R/T yelling accusations C. Social isolation R/T paranoia AEB absence from classes D. Risk for self-directed violence R/T depressed mood

B (The nursing diagnosis that must be prioritized in this situation should be risk for other-directed violence R/T yelling accusations. Hearing voices and yelling accusations indicates a potential for violence, and this potential safety issue should be prioritized.)

Which statement accurately differentiates NCD from pseudodementia (depression)? A: NCD has a rapid onset, whereas pseudodementia does not. B: NCD symptoms include disorientation to time and place, and pseudodementia does not. C: NCD symptoms improve as the day progresses, but symptoms of pseudodementia worsen. D: NCD causes decreased appetite, whereas pseudodementia does not.

B: NCD symptoms include disorientation to time and place, and pseudodementia does not.

A nurse counsels the family of a patient diagnosed with Alzheimer's disease who lives at home and wanders at night. Which action is most important for the nurse to recommend to enhance safety? A: Apply a medical alert bracelet to the patient B: Place locks at the tops of doors C: Discourage daytime napping D: Obtain a bed with side rails

B: Place locks at the tops of doors

Which assessment finding would be likely for a patient experiencing a hallucination? The patient: A: Looks at shadows on a wall and says, "I see scary faces". B: States, "I feel bugs crawling on my legs and biting me". C: Reports telepathic messages from the television. D: Speaks in rhymes

B: States, "I feel bugs crawling on my legs and biting me".

A client with a history of cerebrovascular accident (CVA) is brought to an emergency department experiencing memory problems, confusion, and disorientation. Based on this client's assessment data, which diagnosis would the nurse expect the physician to assign? A: Delirium due to adverse effects of cardiac medications. B: Vascular neurocognitive disorder C: Altered thought processes D: Alzheimer's disease

B: Vascular neurocognitive disorder

A newly admitted client has taken thioridazine (Mellaril) for 2 years with good symptom control. Symptoms exhibited on admission included paranoid delusions and hallucinations. The nurse should recognize which potential cause for the return of these symptoms? A. The client has developed tolerance to the antipsychotic medication. B. The client has not taken the medication with food. C. The client has not taken the medication as prescribed. D. The client has combined alcohol with the medication.

C (Altered thinking can affect a client's insight into the necessity for taking antipsychotic medications consistently. When symptoms are no longer bothersome, clients may stop taking medications that cause disturbing side effects. Clients may miss the connection between taking the medications and an improved symptom profile.)

A client states, "I hear voices that tell me that I am evil." Which outcome related to these symptoms should the nurse expect this client to accomplish by discharge? A. The client will verbalize the reason the voices make derogatory statements. B. The client will not hear auditory hallucinations. C. The client will identify events that increase anxiety and illicit hallucinations. D. The client will positively integrate the voices into the client's personality structure.

C (It is unrealistic to expect the client to completely stop hearing voices. Even when compliant with antipsychotic medications, clients may still hear voices. It would be realistic to expect the client to associate stressful events with an increase in auditory hallucinations. By this recognition the client can anticipate symptoms and initiate appropriate coping skills.)

A client diagnosed with schizophrenia states, "Can't you hear him? It's the devil. He's telling me I'm going to hell." Which is the most appropriate nursing reply? A. "Did you take your medicine this morning?" B. "You are not going to hell. You are a good person." C. "I'm sure the voices sound scary. The devil is not talking to you. This is part of your illness." D. "The devil only talks to people who are receptive to his influence."

C (The most appropriate reply by the nurse is to reassure the client with an accepting attitude while not reinforcing the hallucination. Reminding the client that "the voices" are a part of his or her illness is a way to help the client accept that the hallucinations are not real.)

Parents ask a nurse how they should reply when their child, diagnosed with paranoid schizophrenia, tells them that voices command him to harm others. Which is the appropriate nursing reply? A. "Tell him to stop discussing the voices." B. "Ignore what he is saying, while attempting to discover the underlying cause." C. "Focus on the feelings generated by the hallucinations and present reality." D. "Present objective evidence that the voices are not real."

C (The most appropriate response by the nurse is to instruct the parents to focus on the feelings generated by the hallucinations and present reality. The parents should maintain an attitude of acceptance to encourage communication but should not reinforce the hallucinations by exploring details of content. It is inappropriate to present logical arguments to persuade the client to accept the hallucinations as not real.)

An aging client diagnosed with chronic schizophrenia takes an antipsychotic and a beta-adrenergic blocking agent (propranolol) for hypertension. Understanding the combined side effects of these drugs, which statement by a nurse is most appropriate? A. "Make sure you concentrate on taking slow, deep, cleansing breaths." B. "Watch your diet and try to engage in some regular physical activity." C. "Rise slowly when you change position from lying to sitting or sitting to standing." D. "Wear sunscreen and try to avoid midday sun exposure."

C (The most appropriate statement by the nurse is to instruct the client to rise slowly when changing positions. Antipsychotic medications and beta blockers cause a decrease in blood pressure. When given in combination, this side effect places the client at risk for developing orthostatic hypotension.)

A client diagnosed with paranoid schizophrenia states, "My psychiatrist is out to get me. I'm sad that the voice is telling me to stop him." What symptom is the client exhibiting, and what is the nurse's legal responsibility related to this symptom? A. Magical thinking; administer an antipsychotic medication B. Persecutory delusions; orient the client to reality C. Command hallucinations; warn the psychiatrist D. Altered thought processes; call an emergency treatment team meeting

C (The nurse should determine that the client is exhibiting command hallucinations. The nurse's legal responsibility is to warn the psychiatrist of the potential for harm. A client who is demonstrating a risk for violence could potentially become physically, emotionally, and/or sexually harmful to others or to self.)

A client diagnosed with schizophrenia is slow to respond and appears to be listening to unseen others. Which medication should a nurse expect a physician to order to address this type of symptom? A. Haloperidol (Haldol) to address the negative symptom B. Clonazepam (Klonopin) to address the positive symptom C. Risperidone (Risperdal) to address the positive symptom D. Clozapine (Clozaril) to address the negative symptom

C (The nurse should expect the physician to order risperidone (Risperdal) to address the positive symptoms of schizophrenia. Risperidone (Risperdal) is an atypical antipsychotic used to reduce positive symptoms, including disturbances in content of thought (delusions), form of thought (neologisms), or sensory perception (hallucinations).)

A client diagnosed with schizoaffective disorder is admitted for social skills training. Which information should be taught by the nurse? A. The side effects of medications B. Deep breathing techniques to decrease stress C. How to make eye contact when communicating D. How to be a leader

C (The nurse should plan to teach the client how to make eye contact when communicating. Social skills, such as making eye contact, can assist clients in communicating needs and maintaining connectedness.)

A client diagnosed with psychosis NOS (not otherwise specified) tells a nurse about voices telling him to kill the president. Which nursing diagnosis should the nurse prioritize for this client? A. Disturbed sensory perception B. Altered thought processes C. Risk for violence: directed toward others D. Risk for injury

C (The nurse should prioritize the diagnosis risk for violence: directed toward others. A client who hears voices telling him to kill someone is at risk for responding and reacting to the command hallucination. Other risk factors for violence include aggressive body language, verbal aggression, catatonic excitement, and rage reactions.)

A client is diagnosed with schizophrenia. A physician orders haloperidol (Haldol) 50 mg bid, benztropine (Cogentin) 1 mg prn, and zolpidem (Ambien) 10 mg HS. Which client behavior would warrant the nurse to administer benztropine? A. Tactile hallucinations B. Tardive dyskinesia C. Restlessness and muscle rigidity D. Reports of hearing disturbing voices

C (The symptom of tactile hallucinations and reports of hearing disturbing voices would be addressed by an antipsychotic medication such as haloperidol. Tardive dyskinesia, a potentially irreversible condition, would warrant the discontinuation of an antipsychotic medication such as haloperidol. An anticholinergic medication such as benztropine would be used to treat the extrapyramidal symptoms of restlessness and muscle rigidity.)

A client diagnosed with schizophrenia takes an antipsychotic agent daily. Which assessment finding should a nurse immediately report to the client's attending psychiatrist? A. Respirations of 22 beats/minute B. Weight gain of 8 pounds in 2 months C. Temperature of 104F (40C) D. Excessive salivation

C (When assessing a client diagnosed with schizophrenia who takes an antipsychotic agent daily, the nurse should immediately address a temperature of 104F (40C). A temperature this high can be a symptom of the rare but life-threatening neuroleptic malignant syndrome.)

After one week of continuous mental confusion, an older African American client is admitted with a preliminary diagnosis of AD. What should cause the nurse to question this diagnosis? A: AD does not typically occur in African American clients. B: The symptoms presented are more indicative of Parkinsonism. C: AD does not develop suddenly D: There has been no T3- or T4- level evaluation ordered.

C: AD does not develop suddenly

An older adult was stopped by police for driving through a red light. When asked for a driver's license, the adult hands the police officer a pair of sunglasses. What sign of dementia is evident? A: Aphasia B: Apraxia C: Agnosia D: Anhedonia

C: Agnosia

A nurse gives anticipatory guidance to the family of a patient diagnosed with stage 3, mild cognitive decline Alzheimer's disease. Which problem common to that stage should the nurse address? a. Violent outbursts b. Emotional disinhibition c. Communication deficits d. Inability to feed or bathe self

C: Communication deficits

Which symptom should a nurse identify that differentiates clients diagnosed with NCDs from clients diagnosed with mood disorders? A: Altered sleep B: Altered concentration C: Impaired memory D: Impaired psychomotor activity

C: Impaired memory

Which statement accurately differentiates mild NCD from major NCD? A: Major NCD involves disorientation that develops suddenly, whereas mild NCD develops more slowly B: Major NCD involves impairment of abstract thinking and judgment, whereas mild NCD does not. C: Major NCD criteria requires substantial cognitive decline from a previous level of performance, and mild NCD requires modest decline. D: Major NCD criteria requires decline from a previous level of performance in three of the listed domains, and mild NCD requires only one.

C: Major NCD criteria requires substantial cognitive decline from a previous level of performance, and mild NCD requires modest decline.

Which medication prescribed to patients diagnosed with Alzheimer's disease antagonizes N-Methyl-D-Aspartate (NMDA) channels rather than cholinesterase? A: Donepezil (Aricept) B: Rivastigmine (Exelon) C: Memantine (Namenda) D: Galantamine (Razadyne)

C: Memantine (Namenda)

An older adult drove to a nearby store but was unable to remember how to get home or state an address. When police intervened, they found that this adult was wearing a heavy coat and hat, even though it was July. Which stage of Alzheimer's disease is evident? A: Preclinical Alzheimer's disease B: Mild cognitive decline C: Moderately severe cognitive decline D: Severe cognitive decline

C: Moderately severe cognitive decline

A client diagnosed with NCD is disorientated and ataxic and wanders. Which is the priority nursing diagnosis? A: Disturbed thought processes B: Self-care deficit C: Risk for injury D: Altered health-care maintenance

C: Risk for injury

A client diagnosed with AD exhibits progressive memory loss, diminished cognitive functioning, and verbal aggression upon experiencing frustration. Which nursing intervention is most appropriate? A: Organize a group activity to present reality B: Minimize environmental lighting C: Schedule structured daily routines D: Explain the consequences for aggressive behaviors

C: Schedule structured daily routines

Two patients in a residential care facility have dementia. One shouts to the other, "Move along, you're blocking the road." The other patient turns, shakes a fist, and shouts, "You're trying to steal my car." What is the nurse's best action? a. Administer one dose of an antipsychotic medication to both patients. b. Reinforce reality. Say to the patients, "Walk along in the hall. This is not a traffic intersection." c. Separate and distract the patients. Take one to the day room and the other to an activities area. d. Step between the two patients and say, "Please quiet down. We do not allow violence here."

C: Separate and distract the patients. Take one to the day room and the other to an activities area.

A patient with fluctuating levels of awareness, confusion, and disturbed orientation shouts, "Bugs are crawling on my legs. Get them off!" Which problem is the patient experiencing? a. Aphasia b. Dystonia c. Tactile hallucinations d. Mnemonic disturbance

C: Tactile hallucinations

Which nursing diagnoses are most application for a patient diagnosed with severe Alzheimer's disease? A: Acute confusion B: Anticipatory grieving C: Urinary incontinence D: Disturbed sleep pattern E: Risk for caregiver role strain

C: Urinary incontinence D: Disturbed sleep pattern E: Risk for caregiver

A client is diagnosed in stage seven of AD. To address the client's symptoms, which nursing intervention should take priority? A: Improve cognitive status by encouraging involvement in social activities. B: Decrease social isolation by providing group therapies C: promote dignity by providing comfort, safety, and self-care measures D: Facilitate communication by providing assistive devices.

C: promote dignity by providing comfort, safety, and self-care measures

An older patient with severe, late-stage dementia no longer recognizes family members. The family asks how long it will be before this patient recognizes them when they visit. What is the nurse's best reply?

D: "It is disappointing when someone you love no longer recognizes you"

A patient diagnosed with Alzheimer's disease calls the fire department saying, "My smoke detectors are going off." Firefighters investigate and discover that the patient misinterpreted the telephone ringing. Which problem is this patient experiencing? A: Hyperorality B: Aphasia C: Apraxia D: Agnosia

D: Agnosia

A client diagnosed with AD has impairments of memory and judgment and is incapable of performing activities of daily living. Which nursing intervention should take priority? A: Present evidence of objective reality to improve cognition. B: Design a bulletin board to represent the current season C: Label the client's room with name and number. D: Assist with bathing and toileting.

D: Assist with bathing and toileting.

What is the priority need for a patient with late-stage dementia? A: Promotion of self-care activities B: Meaningful verbal communication C: Preventing the patient from wandering D: Maintenance of nutrition and hydration

D: Maintenance of nutrition and hydration

Which is the reason for the proliferation of the diagnosis of NCDs? A: increased numbers of neurotransmitters has been implicated in the proliferation of NCD. B: Similar symptoms of NCD and depression lead to misdiagnoses, increasing numbers of NCD. C: Societal stress contributes to the increase in this diagnosis D: More people now survive into the high-risk period for neurocognitive disorders.

D: More people now survive into the high-risk period for neurocognitive disorders.

An older client has recently moved to a nursing home. the client has trouble concentrating and socially isolates. A physician believes the client would benefit from medication therapy. Which medication should the nurse expect the physician to prescribe? A: Haloperidol (Haldol) B: Donepezil (Aricept) C: Diazepam (Valium) D: Sertraline (Zoloft)

D: Sertraline (Zoloft)

A client diagnosed with AD can no longer ambulate, does not recognize family members, and communicates with agitated behaviors and incoherent verbalizations. The nurse recognizes these symptoms as indicative of which stage of illness? A: Stage 4: Mild-to-Moderate cognitive decline B: Stage 5: Moderate Cognitive decline C: Stage 6: Moderate-to-Severe cognitive decline D: Stage 7: Severe Cognitive decline

D: Stage 7: Severe Cognitive decline

A client diagnosed with vascular neurocognitive disorder (NCD) is discharged to home under the care of his wife. Which information should cause the nurse to question the client's safety? A: His wife works from home in telecommunication B: The client has worked the nightshift his entire career C: His wife has minimal family support D: The client smokes one pack of cigarettes per day.

D: The client smokes one pack of cigarettes per day

Which nursing intervention would be most appropriate when caring for an acutely agitated client diagnosed with paranoid schizophrenia? A. Provide neon lights and soft music. B. Maintain continual eye contact throughout the interview. C. Use therapeutic touch to increase trust and rapport. D. Provide personal space to respect the client's boundaries.

D (The most appropriate nursing intervention is to provide personal space to respect the client's boundaries. Providing personal space may serve to reduce anxiety and thus reduce the client's risk for violence.)

A nurse is assessing a client diagnosed with paranoid schizophrenia. The nurse asks the client, "Do you receive special messages from certain sources, such as the television or radio?" Which potential symptom of this disorder is the nurse assessing? A. Thought insertion B. Paranoid delusions C. Magical thinking D. Delusions of reference

D (The nurse is assessing for the potential symptom of delusions of reference. A client who believes that he or she receives messages through the radio is experiencing delusions of reference. When a client experiences these delusions, he or she interprets all events within the environment as personal references.)

A client diagnosed with chronic schizophrenia presents in an emergency department (ED) with uncontrollable tongue movements, stiff neck, and difficulty swallowing. The nurse would expect the physician to recognize which condition and implement which treatment? A. Neuroleptic malignant syndrome and treat by discontinuing antipsychotic medications B. Agranulocytosis and treat by administration of clozapine (Clozaril) C. Extrapyramidal symptoms and treat by administration of benztropine (Cogentin) D. Tardive dyskinesia and treat by discontinuing antipsychotic medications

D (The nurse should expect that an ED physician would diagnose the client with tardive dyskinesia and discontinue antipsychotic medications. Tardive dyskinesia is a condition of abnormal involuntary movements of the mouth, tongue, trunk, and extremities that can be an irreversible side effect of typical antipsychotic medications.)

During an admission assessment, a nurse notes that a client diagnosed with schizophrenia has allergies to penicillin, prochlorperazine (Compazine), and bee stings. Based on this assessment data, which antipsychotic medication would be contraindicated? A. Haloperidol (Haldol), because it is used only in elderly patients B. Clozapine (Clozaril), because of a cross-sensitivity to penicillin C. Risperidone (Risperdal), because it exacerbates symptoms of depression D. Thioridazine (Mellaril), because of cross-sensitivity among phenothiazines

D (The nurse should know that thioridazine (Mellaril) would be contraindicated because of cross-sensitivity among phenothiazines. Prochlorperazine (Compazine) and thioridazine are both classified as phenothiazines.)

A patient with fluctuating levels of consciousness, disturbed orientation, and perceptual alteration begs, "Someone get these bugs off me." What is the nurse's best response? a. "No bugs are on your legs. You are having hallucinations." b. "I will have someone stay here and brush off the bugs for you." c. "Try to relax. The crawling sensation will go away sooner if you can relax." d. "I don't see any bugs, but I can tell you are frightened. I will stay with you."

D. "I don't see any bugs, but I can tell you are frightened. I will stay with you".

A patient with stage 3 Alzheimer's disease tires easily and prefers to stay home rather than attend social activities. The spouse does the grocery shopping because the patient cannot remember what to buy. Which nursing diagnosis applies at this time? a. Self-care deficit b. Impaired memory c. Caregiver role strain d. Adult failure to thrive

b. impaired memory


Set pelajaran terkait

Chapter 13; correlation and linear regression

View Set

practice questions for med surg II final

View Set

Sleep and Sleep Disorder Ch. 1-4

View Set

Decimals in Standard, Written, and Expanded Forms

View Set